Interesting Properties related to Four 9-point Centers
by TelvCohl, Jul 17, 2016, 12:36 PM
Preliminaries
Lemma 1 (Gergonne-Euler theorem): Given a
and a point
Let
be the reflection of
in
respectively. Then ![$$ \boxed{\frac{[\triangle P_aP_bP_c]}{[\triangle ABC]} = \left | \frac{\text{Power} (P, \odot (ABC))}{{R}^2} \right |} . $$](//latex.artofproblemsolving.com/3/d/c/3dc4ac3c1023edca39b3d753cb02421212751165.png)
Proof : Let
be the orthocenter of
and let
be the second intersection of
with
respectively. Clearly,
is the circumcenter of
so
hence we get
Notice
(from symmetry) and
we get
Similarly, we can prove
and
so we conclude that ![$$ \frac{[\triangle P_aP_bP_c]}{[\triangle ABC]} = \frac{\sum [\triangle HP_bP_c]}{[\triangle ABC]} = \left| \frac{\text{Power}(P,\odot(ABC))}{2} \right | \cdot \frac{\sum \left| \sin 2\angle A \right|}{[\triangle ABC]} = \left|\frac{\text{Power} (P, \odot (ABC))}{{R}^2}\right|. \qquad \blacksquare $$](//latex.artofproblemsolving.com/1/1/8/118513a5af93c98cd183fae8fd6e1c00d222e139.png)
Lemma 2 (Gergonne-Ann theorem) : Given a
with points
Let
be the points such that
and
Then ![$$ \boxed{{[\triangle ABC]}^2 = [\triangle DEF] \cdot [\triangle XYZ ]} .$$](//latex.artofproblemsolving.com/5/2/b/52b75707ae8ca75911a7eda1780510a8c962cd25.png)
Proof : Clearly,
and
are homothetic, so
are concurrent at
Let
(
). Since
so we conclude that ![$$ \frac{[\triangle ABC]}{[\triangle DEF]} = \varsigma = \sqrt{\frac{[\triangle XYZ]}{[\triangle DEF]}} \Longrightarrow {[\triangle ABC]}^2 = [\triangle DEF] \cdot [\triangle XYZ ]. \qquad \blacksquare $$](//latex.artofproblemsolving.com/c/a/f/caf0fd8292448982a239900560bbc19e1a7ca2f1.png)
Lemma 3 : Given a
and a point
Let
be the image of
under the Inversion WRT
and let
be the pedal triangle of
WRT
respectively. Let
be the midpoint of
and let
be the isogonal conjugate of
WRT
Then

Proof : Let
be the reflection of
in
respectively. Since
and
are inversely similar, so
hence notice that
we get
On the other hand, since
is the A-apollonius circle of
so
and
are directly similar
Analogously, we can prove
and
so we conclude that 
Lemma 4 : Given a
and a point
Let
be the pedal triangle of
WRT
and let
be the circumcenter of
respectively. Let
be the isogonal conjugate of
WRT
respectively and let
be the Poncelet point of
respectively. Then the Steiner line of
WRT
is parallel to
respectively.
Proof : Let
be the pedal triangle of
WRT
and let
be the projection of
on
respectively. For any point
lying on the circumcircle of the triangle
let
be the Steiner line of
WRT
Since
are symmetry WRT the A-midline of
(well-known), so notice
we get 
Let
be the circumcevian triangle of
WRT
respectively. It's obvious that
so notice
we conclude that 
Main result
Notation (all symbols in this section bear the same meaning)
Let
be the isogonal conjugate of
Let
be the 9-point center of
respectively. Let
be the pedal triangle of
WRT
and let
be the isogonal conjugate of
WRT
Let
be the midpoint of
(center of the pedal circle of
WRT
).
Property 1 :
and
are inversely similar.
Proof : Let
be the midpoint of
respectively. Since
are concurrent at the Poncelet point
of
so
Analogously, we can prove
and
so

Let
be the pedal triangle of
WRT
Since
(
)
so
are isogonal conjugate WRT
Similarly, we can prove
are isogonal conjugate WRT
so
are isogonal conjugate WRT

Property 2 : Let
be the circumcenter of
then
and ![$ \frac{QQ^*}{2OP} =\frac{[\triangle Q_aQ_bQ_c]}{[\triangle ABC]}. $](//latex.artofproblemsolving.com/7/a/8/7a8aad2ea62b35a1bca84b1973da482f69f6a4f8.png)
Proof : Let
be the pedal triangle of
WRT
and let
be its circumcenter (midpoint of
). From
we get
so combining
we get
and
are homothetic
From Lemma 2
so we conclude that ![$$ \frac{QQ^*}{2OP} = \frac{JQ^*}{OP} = \sqrt{\frac{[\triangle Q_a^*Q_b^*Q_c^*]}{[\triangle ABC]}} = \frac{[\triangle Q_aQ_bQ_c]}{[\triangle ABC]}. \qquad \blacksquare $$](//latex.artofproblemsolving.com/d/0/e/d0eb904c325e27fe96ea00ea892dcbbff434d044.png)
Property 3 : Let
be the image of
under the Inversion WRT
and let
be the pedal triangle of
WRT
Then
are homothetic with homothety center 
Proof : Since the image of
under the homothety
are concurrent at the antigonal conjugate
of
WRT
which is also the isogonal conjugate of
WRT
(well-known), so
(
),
(
),
(
)
and
are homothetic. Let
be the midpoint of
respectively. Obviously,
and
are homothetic. Since
so combining Lemma 1 and Property 2 we get ![$$ \frac{RQ}{RO} = \frac{ \left |{R}^2-{OQ}^2 \right |}{{R}^2} = \frac{4[\triangle Q_aQ_bQ_c]}{[\triangle ABC ]} = \frac{QQ^*}{OW} \Longrightarrow Q^* \in WR . $$](//latex.artofproblemsolving.com/5/b/3/5b3daa11b8c40a007a286e141648fa2cafee91d7.png)
On the other hand, from
we know
lies on
so combining
(Property 1 + Lemma 3) we conclude that
is the homothety center of
and

Property 4 :
is the image of
under the Inversion WRT 
Proof : Let
be the radius of
Since
and the circumcevian triangle of
WRT
are directly similar with corresponding point
so
hence we conclude that 
Property 5 : Let
be the center of
respectively. Then the harmonic conjugate
of
WRT
is the homothety center of of
and

Proof : Since
is the reflection of
in
respectively, so we conclude that 
Corollary 6 : Let
be the center of
respectively. Then
is the homothety center of
and

Property 7 : Let
be the isogonal conjugate of
WRT
Then
lies on
and 
Proof : Since
so
lies on
Similarly, we can prove

Property 8 :
and of
share the same orthocenter 
Proof : This is a particular case of Interesting Hexagon (Property 6).
Property 9 : The circumcenter
of
is the complement (WRT
) of the 9-point center
of the antipedal triangle
of
WRT 
Proof : Let
be the circumcenter of
respectively. Since
is the reflection of
in the midpoint of
respectively, so the centroid
of
is the midpoint of the centroid of
is the centroid of
hence the 9-point center
of
is the complement of
WRT
Since
is the midpoint of
and
is the reflection of
in
so from Menelaus' theorem (
and
) we conclude that
is the anticomplement of
WRT

Property 10 :
lies on 
Proof : Let
be the circumcenter of the antipedal triangle of
WRT
From Property 4
the reflection of
in
is the image of
under the inversion WRT
so
lies on
the midpoint
of
are collinear. 
Corollary 11 : The circumcenter
of
lies on
Furthermore,
Property 12 : Let
be the radius of
respectively. Then

Proof : From symmetry, it suffices to prove
Since the anticomplement
of
WRT
is the pole of the Simson line of
with direction
so from Lemma 4 we know
is the orthopole of
WRT
hence
lies on the complement of
WRT
Combining Property 9
is the complement of
WRT
so we conclude that 
Lemma 1 (Gergonne-Euler theorem): Given a









![$$ \boxed{\frac{[\triangle P_aP_bP_c]}{[\triangle ABC]} = \left | \frac{\text{Power} (P, \odot (ABC))}{{R}^2} \right |} . $$](http://latex.artofproblemsolving.com/3/d/c/3dc4ac3c1023edca39b3d753cb02421212751165.png)
Proof : Let





















![$ [\triangle HP_bP_c] $](http://latex.artofproblemsolving.com/5/2/5/5259c145920a2ef53481614c78e1e8ba24ef531f.png)

![$ [\triangle EP_bP_c] $](http://latex.artofproblemsolving.com/6/3/a/63a778b92ecaf49c580c17e3e3da1d1b8587ee2f.png)


















![$$ [\triangle HP_bP_c] = \left| \frac{AP \cdot PD}{2} \right| \cdot \left| \sin 2\angle A \right| = \left| \frac{\text{Power}(P,\odot(ABC))}{2} \right| \cdot \left| \sin 2\angle A \right| . $$](http://latex.artofproblemsolving.com/6/f/8/6f87c1f818159c7483a71281626a0ae12f27f4fe.png)
![$ [\triangle HP_cP_a] $](http://latex.artofproblemsolving.com/b/3/c/b3cae529251dc30dd06e67326420709b849cae36.png)




![$ [\triangle HP_aP_b] $](http://latex.artofproblemsolving.com/5/9/3/593a996c22dac6effc0b4165288bc164ee7753a6.png)




![$$ \frac{[\triangle P_aP_bP_c]}{[\triangle ABC]} = \frac{\sum [\triangle HP_bP_c]}{[\triangle ABC]} = \left| \frac{\text{Power}(P,\odot(ABC))}{2} \right | \cdot \frac{\sum \left| \sin 2\angle A \right|}{[\triangle ABC]} = \left|\frac{\text{Power} (P, \odot (ABC))}{{R}^2}\right|. \qquad \blacksquare $$](http://latex.artofproblemsolving.com/1/1/8/118513a5af93c98cd183fae8fd6e1c00d222e139.png)
Lemma 2 (Gergonne-Ann theorem) : Given a































![$$ \boxed{{[\triangle ABC]}^2 = [\triangle DEF] \cdot [\triangle XYZ ]} .$$](http://latex.artofproblemsolving.com/5/2/b/52b75707ae8ca75911a7eda1780510a8c962cd25.png)
Proof : Clearly,








![$ [\triangle DEF] $](http://latex.artofproblemsolving.com/a/e/f/aefa5b8fbc713981197c2431a44945c6f3b0a262.png)

![$ [\triangle XYZ] $](http://latex.artofproblemsolving.com/f/8/2/f82a02bf4b89f297db88554d452816278819e907.png)



![$$ \frac{[AETF]}{[\triangle TEF]} = \frac{\text{dist}(T,YZ)}{\text{dist}(T,EF)} = \varsigma, \frac{[BFTD]}{[\triangle TFD]} = \frac{\text{dist}(T,ZX)}{\text{dist}(T,FD)} = \varsigma, \frac{[CDTE]}{[\triangle TDE]} = \frac{\text{dist}(T,XY)}{\text{dist}(T,DE)} = \varsigma, $$](http://latex.artofproblemsolving.com/8/8/a/88a59856bae5472327987f7150a054bbdf0f440a.png)
![$$ \frac{[\triangle ABC]}{[\triangle DEF]} = \varsigma = \sqrt{\frac{[\triangle XYZ]}{[\triangle DEF]}} \Longrightarrow {[\triangle ABC]}^2 = [\triangle DEF] \cdot [\triangle XYZ ]. \qquad \blacksquare $$](http://latex.artofproblemsolving.com/c/a/f/caf0fd8292448982a239900560bbc19e1a7ca2f1.png)
Lemma 3 : Given a






















Proof : Let


























































Lemma 4 : Given a























Proof : Let




















Let





















Main result
Notation (all symbols in this section bear the same meaning)
Let






















Property 1 :










Proof : Let

































Let




































Property 2 : Let





![$ \frac{QQ^*}{2OP} =\frac{[\triangle Q_aQ_bQ_c]}{[\triangle ABC]}. $](http://latex.artofproblemsolving.com/7/a/8/7a8aad2ea62b35a1bca84b1973da482f69f6a4f8.png)
Proof : Let















































![$ [\triangle Q_a^*Q_b^*Q_c^*] $](http://latex.artofproblemsolving.com/0/f/e/0febeff8568b3fd4b90905fb64385ac20770dc35.png)

![$ [\triangle ABC ] $](http://latex.artofproblemsolving.com/1/4/4/1443374b64a8b33588a803f24987b9f8e46e0e5a.png)

![$ {[\triangle Q_aQ_bQ_c]}^2, $](http://latex.artofproblemsolving.com/5/1/3/51361f78615ab5c15305e32f04050a402ca7db98.png)
![$$ \frac{QQ^*}{2OP} = \frac{JQ^*}{OP} = \sqrt{\frac{[\triangle Q_a^*Q_b^*Q_c^*]}{[\triangle ABC]}} = \frac{[\triangle Q_aQ_bQ_c]}{[\triangle ABC]}. \qquad \blacksquare $$](http://latex.artofproblemsolving.com/d/0/e/d0eb904c325e27fe96ea00ea892dcbbff434d044.png)
Property 3 : Let









Proof : Since the image of









































![$$ \frac{RQ}{RO} = \frac{ \left |{R}^2-{OQ}^2 \right |}{{R}^2} = \frac{4[\triangle Q_aQ_bQ_c]}{[\triangle ABC ]} = \frac{QQ^*}{OW} \Longrightarrow Q^* \in WR . $$](http://latex.artofproblemsolving.com/5/b/3/5b3daa11b8c40a007a286e141648fa2cafee91d7.png)
On the other hand, from


























Property 4 :



Proof : Let











Property 5 : Let
















Proof : Since








Corollary 6 : Let













Property 7 : Let






Proof : Since











Property 8 :



Proof : This is a particular case of Interesting Hexagon (Property 6).

Property 9 : The circumcenter







Proof : Let



































Property 10 :


Proof : Let


















Corollary 11 : The circumcenter









Property 12 : Let










Proof : From symmetry, it suffices to prove























